Quore’s production

This topic has expert replies
Master | Next Rank: 500 Posts
Posts: 117
Joined: Mon Oct 27, 2008 5:08 pm
Thanked: 1 times

Quore’s production

by [email protected] » Sun Feb 22, 2009 7:20 am
25. Because of the recent transformation of the market. Quore, Inc., must increase productivity, 10 percent over the course of the next two years, or it will certainly go bankrupt. In fact, however, Quore’s production structure is such that if a 10 percent productivity increase is possible, then a 20 percent increase is attainable.
If the statements above are true, which one of the following must on the basis of them also be true?
(A) It is only Quore’s production structure that makes it possible for Quore to survive the transformation of the market.
(B) Quore will not go bankrupt if it achieves a productivity increase of 20 percent over the next two years.
(C) If the market had not been transformed, Quore would have required no productivity increase in order to avoid bankruptcy.
(D) Because of the transformation of the market, Quore will achieve a productivity increase of 10 percent over the next two years.
(E) If a 20 percent productivity increase is unattainable for Quore, then it must go bankrupt.


e

Master | Next Rank: 500 Posts
Posts: 174
Joined: Thu Jan 08, 2009 12:04 am
Thanked: 5 times
GMAT Score:620

by bmlaud » Sun Feb 22, 2009 7:44 am
I go with E

The argument says that Company requires to improve the productivity by 10 %(required condition) to avoid bankruptcy(consequence). secondly if 10% increase in productivity is possible then 20% increase is attainable (achievable condition).

Going by the opposite way, 20% increase in productivity is unattainable-- why??? the required condition of 10% increase in productivity is not satisfied-- if the required condition is not satified the consequence (bankruptcy) will occur.

Master | Next Rank: 500 Posts
Posts: 137
Joined: Thu Jan 08, 2009 1:27 am
Thanked: 7 times

by welcome » Mon Feb 23, 2009 8:08 am
IMO E.
Shubham.
590 >> 630 >> 640 >> 610 >> 600 >> 640 >> 590 >> 640 >> 590 >> 590

Master | Next Rank: 500 Posts
Posts: 188
Joined: Sun Dec 23, 2007 7:40 am
Location: INDIA
Thanked: 4 times

by senthil » Sun Mar 01, 2009 1:53 pm
Yes E is the answer and the logic is clearly explained .

WHY it cant be C? PLease explain
Thanks
Senthil

It seems impossible until its done.

Junior | Next Rank: 30 Posts
Posts: 10
Joined: Mon Jun 25, 2012 10:02 am

by ahmed2502 » Sat Jun 30, 2012 7:23 am
bmlaud wrote:I go with E

The argument says that Company requires to improve the productivity by 10 %(required condition) to avoid bankruptcy(consequence). secondly if 10% increase in productivity is possible then 20% increase is attainable (achievable condition).

Going by the opposite way, 20% increase in productivity is unattainable-- why??? the required condition of 10% increase in productivity is not satisfied-- if the required condition is not satified the consequence (bankruptcy) will occur.
I could not understand this. Can someone help ?
It is given "Quore Inc. must increase productivity 10 percent over the course of the next two years or it will certainly go bankrupt" - where from this 20% came ? We need 10% to avoid bankruptcy, it doesn't matter if we attain 10%-15% or 20%.
Again - all other options seem wrong to me.

User avatar
Legendary Member
Posts: 502
Joined: Tue Jun 03, 2008 11:36 pm
Thanked: 99 times
Followed by:21 members

by vk_vinayak » Sat Jun 30, 2012 9:39 am
What is wrong with B?
- VK

I will (Learn. Recognize. Apply)

User avatar
GMAT Instructor
Posts: 578
Joined: Tue Aug 25, 2009 6:00 pm
Thanked: 136 times
Followed by:62 members

by KapTeacherEli » Sat Jun 30, 2012 6:34 pm
[email protected] wrote:25. Because of the recent transformation of the market. Quore, Inc., must increase productivity, 10 percent over the course of the next two years, or it will certainly go bankrupt. In fact, however, Quore�s production structure is such that if a 10 percent productivity increase is possible, then a 20 percent increase is attainable.
If the statements above are true, which one of the following must on the basis of them also be true?
(A) It is only Quore�s production structure that makes it possible for Quore to survive the transformation of the market.
(B) Quore will not go bankrupt if it achieves a productivity increase of 20 percent over the next two years.
(C) If the market had not been transformed, Quore would have required no productivity increase in order to avoid bankruptcy.
(D) Because of the transformation of the market, Quore will achieve a productivity increase of 10 percent over the next two years.
(E) If a 20 percent productivity increase is unattainable for Quore, then it must go bankrupt.


e
Like most inference question, this one is hard to predict--our best bet is to check each answer choice one at a time.

(A) "Only" is the giveaway that this is incorrect. We don't know that Quore will survive the transformation, nor how many factors went into its survival if it does last.
(B) Quore can only survive bankruptcy if it gets 10%, but as we just said, we have no idea what else is required to avoid bankruptcy. We cannot infer this.
(C) We know that the market transformation triggered this economic crisis, but we don't know how or why; it's possible that a natural disaster, war, or international incident could have had the same effect as the new economy. This is not out choice.
(D) This just isn't true; we have no idea if the increase will happen.
(E) The prompt tells us that if Quore attains 10% productivity increase, then 20% is also possible. That means that if 20% is impossible, 10% is likewise out of reach. And if 10% is impossible? Then, per the stimulus, Quore "will certainly" go bankrupt. This must be true, so it is the correct answer.
Eli Meyer
Kaplan GMAT Teacher
Cambridge, MA
www.kaptest.com/gmat

ImageImageImage

Junior | Next Rank: 30 Posts
Posts: 24
Joined: Mon Mar 12, 2012 9:03 am
Thanked: 1 times

by gmat2805 » Sun Jul 01, 2012 5:14 am
I am still confused; how can inability to reach 20% productivity indicate that even 10% is not achievable . Can some one please explain a bit more on E.

User avatar
GMAT Instructor
Posts: 578
Joined: Tue Aug 25, 2009 6:00 pm
Thanked: 136 times
Followed by:62 members

by KapTeacherEli » Sun Jul 01, 2012 7:49 am
gmat2805 wrote:I am still confused; how can inability to reach 20% productivity indicate that even 10% is not achievable . Can some one please explain a bit more on E.
It says so right in the text: if 10% is achievable, then 20% is attainable. That means that if 20% is not attainable, 10% is not attainable.

This is known as the "contrapositive." It's tested much more often in LSAT Logical Reasoning, but it shows up from time to time in GMAT critical reasoning as well. Basically, X --> Y implies Not Y --> Not X.

Here are a few examples:

If you're in New York, you're in the United States.
If you're not in the United States, you're not in New York.

If you are a giraffe, you're a yellow, spotted, four-hoofed mammal.
If you're not a yellow, spotted, four-hoofed mammal, you're not a giraffe.

you cannot succeed in business without a great business plan.
If you succeeded in business, then you had a great business plan.

Hope this helps!
Eli Meyer
Kaplan GMAT Teacher
Cambridge, MA
www.kaptest.com/gmat

ImageImageImage

User avatar
Master | Next Rank: 500 Posts
Posts: 307
Joined: Wed Feb 22, 2012 9:45 pm
Thanked: 12 times
GMAT Score:700

by Gaurav 2013-fall » Sun Jul 01, 2012 8:06 am
KapTeacherEli wrote:
[email protected] wrote:25. Because of the recent transformation of the market. Quore, Inc., must increase productivity, 10 percent over the course of the next two years, or it will certainly go bankrupt. In fact, however, Quore�s production structure is such that if a 10 percent productivity increase is possible, then a 20 percent increase is attainable.
If the statements above are true, which one of the following must on the basis of them also be true?
(A) It is only Quore�s production structure that makes it possible for Quore to survive the transformation of the market.
(B) Quore will not go bankrupt if it achieves a productivity increase of 20 percent over the next two years.
(C) If the market had not been transformed, Quore would have required no productivity increase in order to avoid bankruptcy.
(D) Because of the transformation of the market, Quore will achieve a productivity increase of 10 percent over the next two years.
(E) If a 20 percent productivity increase is unattainable for Quore, then it must go bankrupt.


e
Like most inference question, this one is hard to predict--our best bet is to check each answer choice one at a time.

(A) "Only" is the giveaway that this is incorrect. We don't know that Quore will survive the transformation, nor how many factors went into its survival if it does last.
(B) Quore can only survive bankruptcy if it gets 10%, but as we just said, we have no idea what else is required to avoid bankruptcy. We cannot infer this.
(C) We know that the market transformation triggered this economic crisis, but we don't know how or why; it's possible that a natural disaster, war, or international incident could have had the same effect as the new economy. This is not out choice.
(D) This just isn't true; we have no idea if the increase will happen.
(E) The prompt tells us that if Quore attains 10% productivity increase, then 20% is also possible. That means that if 20% is impossible, 10% is likewise out of reach. And if 10% is impossible? Then, per the stimulus, Quore "will certainly" go bankrupt. This must be true, so it is the correct answer.

Eli,

I have a doubt (see bold part in option E above).Essentially, you are saying that if A then B, therefore if not B then not A. Is this logic right? I am not sure about it.
Let me tell you something you already know. The world ain't all sunshine and rainbows. It is a very mean and nasty place and it will beat you to your knees and keep you there permanently if you let it. You, me, or nobody is gonna hit as hard as life. But it ain't how hard you hit; it's about how hard you can get hit, and keep moving forward. How much you can take, and keep moving forward. That's how winning is done. Now, if you know what you're worth, then go out and get what you're worth. But you gotta be willing to take the hit, and not pointing fingers saying you ain't where you are because of him, or her, or anybody. Cowards do that and that ain't you. You're better than that! (Rocky VI)

User avatar
GMAT Instructor
Posts: 578
Joined: Tue Aug 25, 2009 6:00 pm
Thanked: 136 times
Followed by:62 members

by KapTeacherEli » Sun Jul 01, 2012 8:32 am
Eli,

I have a doubt (see bold part in option E above).Essentially, you are saying that if A then B, therefore if not B then not A. Is this logic right? I am not sure about it.
That's it!
Eli Meyer
Kaplan GMAT Teacher
Cambridge, MA
www.kaptest.com/gmat

ImageImageImage

User avatar
Legendary Member
Posts: 502
Joined: Tue Jun 03, 2008 11:36 pm
Thanked: 99 times
Followed by:21 members

by vk_vinayak » Sun Jul 01, 2012 8:20 pm
Eli,

Sorry to bother you again. But I am still not sure the difference between B and E.

Argument: 10% (20% indirectly) increase attained -> No bankruptcy.
Let's say X = 10% (20% indirectly) increase and Y = No bankruptcy
So, the argument is X->Y.

The only thing we can deduce from this is ~Y -> ~X
ie Bankruptcy -> No 10% (or 20%) increase attained

B. X -> Y
E. ~X -> ~Y

I think B matches the argument very closely. And it also uses conditional 'will' against strong 'must' used in E.

Where am I going wrong?
- VK

I will (Learn. Recognize. Apply)

User avatar
GMAT Instructor
Posts: 578
Joined: Tue Aug 25, 2009 6:00 pm
Thanked: 136 times
Followed by:62 members

by KapTeacherEli » Sun Jul 01, 2012 9:40 pm
vk_vinayak wrote:Eli,

Sorry to bother you again. But I am still not sure the difference between B and E.

Argument: 10% (20% indirectly) increase attained -> No bankruptcy.
Let's say X = 10% (20% indirectly) increase and Y = No bankruptcy
So, the argument is X->Y.

The only thing we can deduce from this is ~Y -> ~X
ie Bankruptcy -> No 10% (or 20%) increase attained

B. X -> Y
E. ~X -> ~Y

I think B matches the argument very closely. And it also uses conditional 'will' against strong 'must' used in E.

Where am I going wrong?
You've got B backwards-- B is Y --> X

The prompt tells us that 20% is necessary for avoiding bankruptcy. That is, in all scenarios where Quore avoids bankruptcy, achieves a 10% and is capable of achieving 20%.

So, No Bankruptcy --> 20% increase is possible.

But B says the opposite: 20% ---> No Bankruptcy. And that can't be properly inferred!

(For example, "If you're in New York, you're in the US" is true, but "If you're in the US, you're in New York is false!")
Eli Meyer
Kaplan GMAT Teacher
Cambridge, MA
www.kaptest.com/gmat

ImageImageImage

User avatar
Legendary Member
Posts: 502
Joined: Tue Jun 03, 2008 11:36 pm
Thanked: 99 times
Followed by:21 members

by vk_vinayak » Sun Jul 01, 2012 9:56 pm
KapTeacherEli wrote:
vk_vinayak wrote:Eli,

Sorry to bother you again. But I am still not sure the difference between B and E.

Argument: 10% (20% indirectly) increase attained -> No bankruptcy.
Let's say X = 10% (20% indirectly) increase and Y = No bankruptcy
So, the argument is X->Y.

The only thing we can deduce from this is ~Y -> ~X
ie Bankruptcy -> No 10% (or 20%) increase attained

B. X -> Y
E. ~X -> ~Y

I think B matches the argument very closely. And it also uses conditional 'will' against strong 'must' used in E.

Where am I going wrong?
You've got B backwards-- B is Y --> X

The prompt tells us that 20% is necessary for avoiding bankruptcy. That is, in all scenarios where Quore avoids bankruptcy, achieves a 10% and is capable of achieving 20%.

So, No Bankruptcy --> 20% increase is possible.

But B says the opposite: 20% ---> No Bankruptcy. And that can't be properly inferred!

(For example, "If you're in New York, you're in the US" is true, but "If you're in the US, you're in New York is false!")
I think I know where I went wrong. I got the argument backwards. Thanks.
- VK

I will (Learn. Recognize. Apply)